Data Insights Ep. 6: Two-Part Analysis -- Quant-based Questions

แชร์
ฝัง
  • เผยแพร่เมื่อ 26 ต.ค. 2024

ความคิดเห็น • 41

  • @saireddy6261
    @saireddy6261 3 หลายเดือนก่อน +3

    This video shows how a calm mind can find elegant solutions to seemingly lengthy questions.

    • @GMATNinjaTutoring
      @GMATNinjaTutoring  3 หลายเดือนก่อน +4

      Thank you again, Sai! You captured the essence of Alex nicely -- he has a calm, elegant mind. And an outstanding beard.
      Have fun studying!

    • @sj19991
      @sj19991 2 หลายเดือนก่อน

      @@GMATNinjaTutoring the beard gives these stats

  • @yajurkapur1051
    @yajurkapur1051 ปีที่แล้ว +5

    Thanks for all the content! It has been great help!!
    I give my GMAT focus on the 16th of November, i would appreciate if you could tell me by when you will upload all the data insights episodes??

  • @bombaybarbell
    @bombaybarbell หลายเดือนก่อน +1

    Thank you for the video series! Got all the 4 questions right :)

    • @GMATNinjaTutoring
      @GMATNinjaTutoring  หลายเดือนก่อน

      Nice work, and thank you so much for watching! ❤️

  • @Conk-bepis
    @Conk-bepis 5 หลายเดือนก่อน +1

    For Q1 - you can use weighted average method as well, and it would hardly take 5 seconds, but can be tricky to use.
    For Q3 - you can form a relationship b/w x and y like you suggested in Q1
    (eg: x = (20-y)/2)
    Now since every option for x is an integer, x needs to be divisible by 2. So plug in y values from options (that make numerator divisible by 2) and get x. Much easier than using even odd logic you suggested ....
    Edit: Thanks for suggesting the weighted average method in Q4, much faster

  • @olegnurmagomedov750
    @olegnurmagomedov750 6 หลายเดือนก่อน +12

    Working on the 4th question, how is a human supposed to solve it in 2 minutes?

    • @filippo_6755
      @filippo_6755 3 หลายเดือนก่อน

      remember that in data insight, you've got a digital calculator which in this case can be much helpful

  • @hesamjafari9738
    @hesamjafari9738 10 หลายเดือนก่อน +3

    For Q4, you just need to find a combination of numbers in the answer choices that completes the equation. ((2X+Y)/3)=0.96

  • @afterburnerfox
    @afterburnerfox ปีที่แล้ว +3

    Hi good folks at GMAT ninja
    Can you please make a video on how do we approach data insights questions, because time management for these questions is waaaaay off for me, even if I get stuff correct. I'm guessing it's the same for many others

  • @niri_fc
    @niri_fc 21 วันที่ผ่านมา

    Hey Alex, thanks for the great video and presentation. I solved Q4, but took too long, do you have any techniques I could use to cut down my time? Thanks in advance.

    • @GMATNinjaTutoring
      @GMATNinjaTutoring  16 วันที่ผ่านมา

      Great question!
      The quickest way to solve this one is to recognize it as a weighted average. In other words, I'm finding the average of two different values, and one of them weighted twice as heavily as the other.
      To give a quick example, consider the average of the numbers 2,5 and 5. I could think of this as the average of 2 and 5, with 5 twice as heavily weighted as 2 (because I have two 5's). So the average should be closer to 5. In fact, it should be twice as close to 5 as it is to 2. What number between 2 and 5 would fit that description? Well, that would be 4.
      Let's use the same process on the gold bars question. The average of two numbers must give 0.96, and one of those two numbers must be twice as close to 0.96 as the other one. The trial and error at this point isn't too bad. Could 0.92 be the closer value? No, then the other value would be 0.08 away from 0.96, and none of the other answers are that far away. Similar logic could dispense with the other answers until we hit 0.98. If that were the closer value, the father one would be 0.04 away from 0.96, and 0.92 fits the bill there.
      So using weighted average logic, the two equally weighted bars must have a weight of 0.98 each, and the other bar must weigh 0.92.
      Thanks again for the question, and let me know if that helps!

  • @ashishsinha9035
    @ashishsinha9035 24 วันที่ผ่านมา

    The last question was very tough !

  • @phanurutpeammetta2066
    @phanurutpeammetta2066 8 หลายเดือนก่อน

    Thank you very much for the great video GMAT Ninja team. I have one question though for Q3. I noticed that apart from x = 8 and y = 4 (8*60 + 4*30 = 600), we can also use x = 6, y = 8 or x = 7, y = 6 to get the same total door prize amount. Could you help clarify why only 8,4 is the correct answer here?

    • @GMATNinjaTutoring
      @GMATNinjaTutoring  8 หลายเดือนก่อน

      While your calculations work out perfectly, neither of the options you provide can be the correct answer because 6 is not an option we can select from the table.
      I hope that helps!

  • @anne-catherinekayser8793
    @anne-catherinekayser8793 7 หลายเดือนก่อน

    For Q3: 2 times an odd number also equals an even number E.g. 2x3= 6

  • @uditdhiman850
    @uditdhiman850 7 หลายเดือนก่อน +1

    Hi Alex, in Q2, how can we cancel "day" with "days" in the denominator when calculating the rate. If there were 5 days, 8 hours per day and 10 workers, won't the worker -hours become = 5*8*10 ? Can you help on this please

    • @popkor7762
      @popkor7762 7 หลายเดือนก่อน

      +1

    • @MrShubhamg
      @MrShubhamg 7 หลายเดือนก่อน

      Correct. Actually answer should be 800/(10*10*10) = 800/1000

    • @popkor7762
      @popkor7762 7 หลายเดือนก่อน

      ​@@MrShubhamg Hi, thanks for the answer. so how should we solve that problem? because we dont know the D(days) x WH (work hours). It's Rate x DaysXwork hours (80) = 400. We have 2 variables so I tried to solve it with the '800' extra information.

    • @MrShubhamg
      @MrShubhamg 7 หลายเดือนก่อน

      @@popkor7762 we have 800 units remaining in 10 days for 10 hours by 10 workers. So 800/(10*10*10)

    • @GMATNinjaTutoring
      @GMATNinjaTutoring  7 หลายเดือนก่อน

      As a starting point, let's take the scenario you suggest: we have 10 workers, each of whom works 8 hours per day. If I multiply those together, I'd get 80 worker-hours per day. Or put another way, I'd have 80 worker-hours/day.
      Now, how many total worker hours would I have after 5 days? Well, as you say, we'd multiply 8 x 10 x 5 = 400 to get that answer. But why does that work? Well, we already established that we have 80 worker-hours/day. If we multiply that unit (worker-hours/day) by days, what would happen? The days would cancel, and we'd be left with just worker-hours.
      For some further explanation of how to work with units (and why "days" cancels in the above expression), you may want to check out our video on rates and unit conversions: th-cam.com/video/ZuZ1J0kXmRw/w-d-xo.html
      I hope that helps!

  • @sebpranavchaudhari
    @sebpranavchaudhari 26 วันที่ผ่านมา

    For Q2 , how can per day cancelled by no of days .?

    • @GMATNinjaTutoring
      @GMATNinjaTutoring  25 วันที่ผ่านมา

      Good question!
      Let's say I'm working 10 hours per day, and I do this for 8 days. How many total hours did I work? Logically, the answer would be 80 hours.
      But how does this work in terms of units? Well, if I multiply 10 (hours/day) x 8 (days), notice that days cancels with "per day" and I get a total of 80 hours worked.
      Thanks for the question, and let me know if that helps!

  • @abhaypratapsingh1260
    @abhaypratapsingh1260 หลายเดือนก่อน

    Hi i still don't get Q1 Like why can't I take $100F borrow from C AND $100 P borrow form C that makes 40%=60$

    • @GMATNinjaTutoring
      @GMATNinjaTutoring  หลายเดือนก่อน

      Good question! Generally speaking, I'd be wary of the "number picking" method. In many cases, the algebraic method will be more efficient. Additionally, sometimes picking numbers can get you into trouble, because in many situations, only certain numbers are valid for that question.
      This question is a good example of that pitfall. In other words, I can't just assume that both Francois and Pierre borrowed $100 from Claudine. Given the constraints of the question, it turns out that Francois must owe 3X the amount that Pierre owes. So if Pierre owes $100, the constraints of the question mean that Francois would have to owe $300. But this logic would be impossible to see without doing the algebra first.
      Thanks for the question, and let me know if that helps at all!

  • @149prajeeshps5
    @149prajeeshps5 หลายเดือนก่อน +1

    Q2 , the solution 400/(8*10), how is that supposed to give us rate of work, instead its giving no.of days right? what am I missing here?

    • @GMATNinjaTutoring
      @GMATNinjaTutoring  หลายเดือนก่อน

      Good question! Notice that the second blank reads "the product of ________ and the number of days since workers began." So the passage is including number of days, and we just need to fill in the missing part. That being said, here is a more complete explanation:
      Consider the equation W = nRT. W = total work; n = # of workers; R = rate of EACH worker individually; T = total time.
      Now, for this question, W= 400, and there were 10 workers, each working 8 hours per day. So R = 400/(10 x 8 x # of days since work began).
      So if we fill in the two blanks with 400 and 80, this should give us R.
      Thanks for the question, and let me know if that helps at all!

    • @DolphRehbein
      @DolphRehbein 7 วันที่ผ่านมา

      @@GMATNinjaTutoring I found the second question difficult as well because you can solve for the rate of CB manufacturing per worker hour with the given information (r=800/1000) assuming that this rate is constant which is implied in the problem. It feels unnatural to 'solve' the problem with unknown information i.e the number of days since the workers began.

  • @ArjunKumar-lt8jd
    @ArjunKumar-lt8jd 10 หลายเดือนก่อน

    What was the difficulty levels of the questions?

    • @GMATNinjaTutoring
      @GMATNinjaTutoring  10 หลายเดือนก่อน +1

      According to mba.com, each of these questions is either a medium or hard difficulty. Overall, I'd consider questions 2 and 4 to be especially challenging.
      Working through these is definitely good practice, but keep in mind -- you don't need to nail every last question on the Data Insights section to get a strong score.

  • @richa30aug
    @richa30aug 11 หลายเดือนก่อน

    .94 and .99 also gives .96 average ? I am doing any mistake

    • @gutstodance9999
      @gutstodance9999 11 หลายเดือนก่อน

      there are 3 elements and not 2

    • @GMATNinjaTutoring
      @GMATNinjaTutoring  11 หลายเดือนก่อน

      Good question!
      Keep in mind we're trying to find the average of three elements here, two of which are the same. Ultimately, their average should give 0.96. So what's the best way to approach this?
      One method is "intelligent" trial and error. Since the average of the three elements is 0.96, we're looking for a total sum of 2.88. Going through the values in the chart strategically shows that 2(0.98) + 1(0.92) = 2.88. Notice that 0.94 and 0.99 would give a sum larger than 2.88, so those couldn't work.
      Another angle is here is to use some "weighted average" logic. In other words, we know that the value 0.96 is more heavily weighted towards the element that appears twice (i.e. the first two bars). Furthermore, since there are twice as many of the first two bars as the third bar, 0.96 needs to be twice as close to the first two bars as the third bar. Looking at our values, the only combination that fits this scenario is 0.98 for the first two bars, and 0.92 for the third bar. Again, 0.94 and 0.99 don't fit this logic (i.e. 0.94 is not twice as close to 0.96 as 0.99), so we know that wouldn't work.
      Thank you for the question, and let me know if that helps clear things up!

  • @Techno34567
    @Techno34567 8 หลายเดือนก่อน

    Great questions as always! I got one more, what's the secret to that beard growth. Thanks!

    • @GMATNinjaTutoring
      @GMATNinjaTutoring  8 หลายเดือนก่อน

      We conducted an internal study, and it turns out that IPA consumption catalyzes beard growth, at least among GMAT tutors. 🤷🏻‍♂️

  • @aseemgarg4553
    @aseemgarg4553 ปีที่แล้ว +1

    250 and 375